LSAT and Law School Admissions Forum

Get expert LSAT preparation and law school admissions advice from PowerScore Test Preparation.

 Administrator
PowerScore Staff
  • PowerScore Staff
  • Posts: 8916
  • Joined: Feb 02, 2011
|
#66013
Complete Question Explanation

The correct answer choice is (D).

Answer choice (A):

Answer choice (B):

Answer choice (C):

Answer choice (D): This is the correct answer choice.

Answer choice (E):


This explanation is still in progress. Please post any questions below!
 hbaezh
  • Posts: 13
  • Joined: Jan 04, 2021
|
#103335
I do not understand why N cannot be 3rd....... P can go on either side of N, so why can't N be in 3rd if it does not restrict the location of P
 Adam Tyson
PowerScore Staff
  • PowerScore Staff
  • Posts: 5153
  • Joined: Apr 14, 2011
|
#103340
Paul's diagrams in the setup thread might help you see the issue with that, hbaezh:

viewtopic.php?f=1305&t=31856

If L is before J, and P needs to be between L and J, then P (and thus the NP rotating block) must be after L and before J. That gives us this diagram:

T - M - L - N/P - J

If you put N 3rd, T and J would be in the first two spaces, and P would be 4th, follow by M and then L. That violates the restriction given in the question, that L is somewhere before J.

Get the most out of your LSAT Prep Plus subscription.

Analyze and track your performance with our Testing and Analytics Package.